LSAT and Law School Admissions Forum

Get expert LSAT preparation and law school admissions advice from PowerScore Test Preparation.

 Administrator
PowerScore Staff
  • PowerScore Staff
  • Posts: 8917
  • Joined: Feb 02, 2011
|
#38547
Complete Question Explanation
(The complete setup for this game can be found here: lsat/viewtopic.php?t=12865)

The correct answer choice is (A)

To attack this question, use proper List question technique: take one rule and apply it to all answer choices, then take another rule, and apply it to the remaining answer choices, and so on, until only one answer choice remains.

Answer choice (A): This is the correct answer choice.


Answer choice (B): This answer choice violates the third rule and is therefore incorrect.

Answer choice (C): This answer choice violates the second rule and is therefore incorrect.

Answer choice (D): This answer choice violates the second rule and is therefore incorrect.

Answer choice (E): This answer choice violates an inference resulting from the combination of the third and fourth rules: when L’s consecutive second shifts fall on Monday and Tuesday, K must work the second shift on Friday. See Template 1.

Get the most out of your LSAT Prep Plus subscription.

Analyze and track your performance with our Testing and Analytics Package.